Jimmy, who is five and a half feet tall, sees a bird at the top of a tree and wonders how tall the tree is. Jimmy stands 18 feet from the tree. Jimmy takes an inclinometer (a device used to calculate angles of elevation) and measures the angle created from his horizontal gaze and the bird to be 50 degrees. How tall is the tree?

Answers

Answer 1

Answer:

100

Step-by-step explanation:

it is 50 when he sees it in a horizontal line so double it to see how much it is if it where straight


Related Questions

8x(x-2017)-2x+4034=0

Answers

Answer:

¼ và 2017

Step-by-step explanation:

What is the leading coefficient of 8 -5x+x^3-2x^4

Answers

Answer:

-2

Step-by-step explanation:

8 -5x+x^3-2x^4

Write the polynomial from highest power of x to lowest power of x

-2x^4+x^3-5x+8

The leading coefficient is the number in front of the highest power of x

-2

root 6 x root 8 is equal to​

Answers

Answer:

[tex]4\sqrt{3}[/tex]

Step-by-step explanation:

[tex]4\sqrt{3}[/tex] = [tex]\sqrt{6}[/tex] x [tex]\sqrt{8}[/tex]

Answer:

4 root 3 is the answer

Step-by-step explanation:

hope its help

If m∠A = m∠B and m∠A + m∠C = m∠D, then
m∠B + m∠C = m∠D.

Answers

Answer:

True. The statement is also equivalent to: If m<B + m<C does not equal m<D, then m<A does not equal m<B and m<A + m<C is not equal to m<D.

Lainey runs a string of lights from the ground straight up a door that is 2.5 m tall. Then he runs the rest of the string in a straight line to a point on the ground which is 6 m from the base of the door frame. There are 10 lights per meter of the string. How many total lights are on the string?

Answers

Answer:

150 because 2.5 × 6 = 15 × 10 = 150

write brackets () in this statement to make it correct 6 + 4 x 5 - 3 = 14

Answers

Answer:

6+4*(5-3)

Step-by-step explanation:

Because of PEMDAS l, we first solve in the parantheses. So 5-3 would be 2, then we multiply 2 by 4 which would be 8 obviously. Then we do the final step, add 6 and 8 which would be 14. Cheers!

Answer: 6 + 4 × (5-3)

Explanation:

6 + 4 × (5-3) = 14

6 + 4 × 2 = 14

6 + 8 = 14

14 = 14

Must click thanks and mark brainliest

What is the answer to this question i really need it asap, thank you.

Answers

Answer:

sorry I could not help you but I wish you luck

Which expressions are equivalent to 2 (5x - 3/4)? click two or more answers

A) -2 (5x) + (-2) (-3/4)
B) -10x - 3/4
C) -10x + 6/2
D) -10x + 3/2
E) -10x - 6/2

Answers

none of them actually. the answer would be 10x - 3/2

please help me is for my homework​

Answers

33.3 (repeating) hope this helps

Answer:

33.33%

Step-by-step explanation:

do divide 3÷9 and that is .333333333

and to convert a decimal to a percent you move the dot two number back so you get 33.33%

PLEASE HELP 25 MINUTES LEFT
38
Marlene wrote an arithmetic sequence where al
=
8 and the common difference is -3.
What is a rule for the nth term of this sequence?

Answers

Answer:

D

Step-by-step explanation:

The nth term of an arithmetic sequence is

[tex]a_{n}[/tex] = a₁ + (n - 1)d

where a₁ is the first term and d the common difference

Here a₁ = 8 and d = - 3 , then

[tex]a_{n}[/tex] = 8 - 3(n - 1) = 8 - 3n + 3 = - 3n + 11 → D

9. The scatter plot below shows the average yearly consumption of
bottled water by people in the United States starting in 1990.
18
16
14
Gallons of
Bottled
Water per
Person
12
10
1990 1992 1994 1996 1998 2000
Year
Using the line of best fit, predict the average consumption of bottled
water in the year 2000?
B) 18 gallons
A) 20 gallons
C) 20 gallons
D) 19 gallons

Answers

A 20 gallons Bb. A Anna

Show ALL STEPS You may use the Balance Strategy Game to help.

x/2+x/4=5

Answers

ohhhh, what? !?? balance ?

How do I solve part c

Answers

Answer:

A possible answer could be 2√2. It depends where points B and C would be. I guess this information was given in the parts A and B.

Step-by-step explanation:

See picture attached

Please help explanation if possible

Answers

Answer:

Step-by-step explanation:

Subtract 3x from both sides

3x - 3x - 2y = -3x + y

Combine

-2y = - 3x + 7  

Divide by - 2

-2y/-2 = -3x/-2 + 7/-2

y = 3x/2 - 7/2

(PICTURE) Im really struggling with questions like these at the moment, if you could please help me out thank you

Answers

Answer:

Arthur should select the Mount Joy Pool when there are less than 10 people at the party

Step-by-step explanation:

The initial fee to rent the Woodbridge Pool = $50

The additional fee per person after renting = $5

The initial fee to rent at Mount Joy Pool = 0 (no initial fee)

The fee per person at Mount Joy Pool = $10

The equation of a straight line is y = m·x + c

Ley y  represent the total cost of renting the pool, x, represent the number of persons in the pool, m represent the fee per person and c represent the initial charges, we get;

The total cost of renting Woodbridge Pool, y = 5·x + 50

The total cost of renting the Joy Pool, y = 10·x

Equating both values of y gives;

5·x + 50 = 10·x

∴ 5·x = 50

x = 50/5 = 10

x = 10

From the above equations, the cost of renting the pool is lower for the Joy Pool when there are less than 10 persons at the pool

It will cost the same amount to rent the pool when the number of persons in the pool are 10 persons and the cost of renting the Mount Joy Pool  will be more than the cost of renting the Woodbridge Pool, when there are more than 10 persons

Therefore, Arthur should select the Mount Joy Pool when there are less than 10 people at the party.

A bus has velocity 20m/s towards east and another bus has velocity 15m/s in West direction.If they start to move from a point simultaneously.What distances do they covered in 2 minutes.​

Answers

Answer:

4,200 m or 4.2 km.

First bus: 2400 m

2nd bus: 1800 m.

Step-by-step explanation:

Their combines speed is 20+15 = 35m/s.

Distance = speed * time

2 minutes = 120 seconds so

D = 35 * 120

= 4200 m.

Splitting the distances:

The first bus covers 20 * 120 = 2400m

The second bus covers 15*120 = 1800m.

Answer:

4200m

Step-by-step explanation:

If two objects A and B are moving in the opposite direction with speed xm/s and ym/s respectively,then;

Relative speed =(x+y)m/s

: Relative speed = 20+15=35m/s

time =2min=160sec

DISTANCE=SPEED ×TIME

D=35×120=4200

4200m

Which inequality is represented by the graph?

Answers

Answer:

It will be -5≤×≤4

I NEED HELP WITH THIS QUESTION ASAP ITS REALLY REALLY URGENT!!!!


Two kids, Albert and Bhara, are 20.0 m apart. Albert sees a soccer ball 25.0 m away. If the angle between the line formed by Albert and Bhara and the line from Albert to the soccer ball is 25 degrees, how far is Bhara from the soccer ball? Correctly round your answer to the nearest tenth of a meter.

Answers

Step-by-step explanation:

I can give you a formula SOH-Sinø=Opposite\Hypotenuse

CAH-Cos=Adjacent\Hypotenuse

TOA-Tan=Opposite\Adjacent

hi, please solve these three questions for me, i have to shoe solving steps.​

Answers

question 3

Step-by-step explanation:

i only able to show you the step of question 3..so sorry

From an 80ft building the angle of elevation of the top of a taller building is 59 degrees and the angle of depression of the base of this building is 65 degrees determine the height of the taller building

Answers

Answer:

First we find the distance between the two buildings =

Step-by-step explanation:

[tex] \frac{80}{ \tan(65) } = 37.3 ft[/tex]

then the upper part of the taller building =

[tex]37.3 \times \tan(59) = 62 \: ft[/tex]

Now the total height of the taller building

[tex]80 + 62 = 142 \: ft[/tex]

help please I'm doing some math homework ​

Answers

Answer:

Slope is 5/2

Step-by-step explanation:

[tex]{ \boxed{ \bf{slope = \frac{y _{2} - y _{1} }{x _{2} - x _{1}} }}}[/tex]

Substitute the terms:

[tex]{ \tt{slope = \frac{7 - ( - 3)}{4 - 0} }} \\ = \frac{5}{2} [/tex]

Five consecutive multiples of 7 have a sum of 350. What is the smallest of these numbers?
A. 70
B. 56
C. 77
D. 84

Answers

Answer:

B. 56

Step-by-step explanation:

x + (x + 7 ) + ( x + 14 ) + ( x + 21 ) + ( x + 28 ) = 350

( x + x + x + x + x ) + ( 7 + 14 + 21 + 28 ) = 350

5x + 70 = 350

     - 70    - 70

_____________

5x = 280

x = 56

Hope this helps!

The smallest number is 56, the correct option is B.

What is Sum?

The sum is the output of the mathematical operation, Addition.

Let the first number is x, they are multiples of 7,

As they are multiples of 7, the consecutive numbers will be added by 7 for next term.

The 5 consecutive numbers can be written as x, (x+7), (x+14), (x+21), (x+28).

The equation can be formed for the numbers that are given as,

An equation is a mathematical statement that relates an algebraic expression with other expression by an equal sign.

The sum of the multiples is 350

x + (x + 7 ) + ( x + 14 ) + ( x + 21 ) + ( x + 28 ) = 350

Grouping the variables and the constants separately

( x + x + x + x + x ) + ( 7 + 14 + 21 + 28 ) = 350

5x + 70 = 350

Adding (-70) to both sides of the equation

5x = 280

Dividing both sides by 5

x = 56

The value of the first number of the series is obtained.

The first value is the smallest number of the series.

To know more about Sum

https://brainly.com/question/13013054

#SPJ2

Write the equation in slope-intercept form. y = 6(x + 2) + 5x

Answers

Answer:

y=11x+12

Step-by-step explanation:

y = 6(x + 2) + 5x

y=6x+12+5x

y=11x+12

in slope interception form=  y=mx+c

                                                y=11x+12

Y=11x + 12 my brother

Will mark brainlest helppppppp​

Answers

Answer:

6

Step-by-step explanation:

again ?

7 = (3x - 4)/2

14 = 3x - 4

18 = 3x

x = 6

A train travels a distance of 60 km at uniform speed. If the speed of the train was reduced by 10 kmh-1, the time taken to travel the 60km will increase by 1/2h. Find the speed of the train at the beginning.

Answers

Answer:

Initial speed is 32 m/s

At uniform speed, acceleration is 0, (a = 0).

When speed reduced, (v - u) = 2.78 ms-¹, t = 1800 sec, s = 60 ,000 metres.

From first equation of motion:

[tex]{ \boxed{ \bf{v = u + at}}} \\ { \tt{(v - u) = at}}[/tex]

substitute:

[tex]{ \tt{2.78 = (a \times 1800)}} \\ { \tt{acceleration = 0.0015 \: {ms}^{ - 2} }}[/tex]

from second equation of motion:

[tex]{ \boxed{ \bf{s = ut + \frac{1}{2} a {t}^{2} }}}[/tex]

substitute:

[tex]{ \tt{60000 = 1800u + ( \frac{1}{2} \times 0.0015 \times {1800}^{2}) }} \\ { \tt{1800u = 57570}} \\ { \tt{u = 32 \: m {s}^{ - 1} }}[/tex]

Math models helpppp plss if you know about math models answer this pls

Answers

Answer:

A mathematical model is a description of a system using mathematical concepts and language. The process of developing a mathematical model is termed mathematical modeling. Mathematical models are used in the natural sciences and engineering disciplines, as well as in non-physical systems such as the social sciences.

Jeanette ice cream shop sold 10 sundaes with nuts and 7 sundaes without nuts to the total number of sundaes

Answers

The answer to the question is 17
There are 10 sundaes with nuts, and 7 without nuts. Our task is to find the total number of sundaes. We can find this by adding 10 + 7. This gives us 17 sundaes.

I suck at math ik but i need anyones help please

Answers

Answer:

f(2)= 1

f-¹(1)= 2

f-¹(f(2))= 2

I hope I helped you^_^

A 60 foot monument casts a 25-foot shadow How long is the shadow of a 4.5 tall person?
A) 1.875 feet (it’s this one)
B) 5.555 feet
C) 2456 feet
D) 13334 feet

Answers

Answer:

A

Step-by-step explanation:

Using similarities we have

(Actual size)/(Shadow length)=60/25=4.5/x

x=4.5*25/60=1.875

Answer:

A) 1.875 feet

Step-by-step explanation:

help me with the picture please

Answers

Answer:  115

=====================================================

Explanation:

Refer to the diagram below. I've drawn diagonal that slopes upward. This diagonal cuts the quadrilateral into two triangles: One is equilateral and the other is isosceles.

The equilateral triangle marked in blue has all three angles 60 degrees each.

Note that the 60 and y angles combined to form 130, so,

60+y = 130

y = 130-60

y = 70

Then focus on the isosceles triangle (angles y, w and w). These three interior angles must add to 180

y+w+w = 180

70+2w = 180

2w = 180-70

2w = 110

w = 110/2

w = 55

This adds onto its adjacent neighbor of 60 to get w+60 = 55+60 = 115 degrees which is the value of x.

Other Questions
Complete the Similarity statement below only if the triangles are similar. please help! Here are the shopping times(in minuts) of nine shoppers at a local grocery store. complete the grouped frequency distribution for the data. in the distribution, the frequency of a class is the number of shopping times in that class.( Note that we are using a class width of 6.) Question 9 of 502 PointsWhat group of people might The Very Young Man represent in The Time Machine?A. PoliticiansB. DoctorsC. TeenagersOD. LawyersReset SelectionDe here to searchWBI70F On his recent free-throw attempts, Lamar made 4 shots and missed 6 shots. Considering thisdata, how many of his next 20 free-throw attempts would you expect Lamar to miss? what is the simplification of 9^8 / 9^7? What are the solutions to the quadratic equation (5y + 6)2 = 24?y = StartFraction negative 6 + 2 StartRoot 6 EndRoot Over 5 EndFraction and y = StartFraction negative 6 minus 2 StartRoot 6 EndRoot Over 5 EndFractiony = StartFraction negative 6 + 2 StartRoot 6 EndRoot Over 5 EndFraction and y = StartFraction 6 minus 2 StartRoot 6 EndRoot Over 5 EndFractiony = StartFraction negative 4 StartRoot 6 EndRoot Over 5 EndFraction and y = StartFraction negative 8 StartRoot 6 EndRoot Over 5 EndFractiony = StartFraction 4 StartRoot 6 EndRoot Over 5 EndFraction and y = StartFraction 8 StartRoot 6 EndRoot Over 5 EndFraction the dog went with thempls rearrange the jumbled sentences If the mass of an object is 15 kg and the velocity is -4 m/s, what is the momentum? c. The reaction Br2 (l) --> Br2 (g) has H = 30.91 kJ/mol and S = 93.3 J/molK. Use this information to show (within close agreement) that the boiling point of bromine is 332 K. Find x (Round to the nearest tenth). PLS HURRY Land surveyors outlined a park as shown. What is the area of the park? Can someone please answer B (PICTURE) [tex]3x + 9 = 12[/tex]Solve for x Bryan works as a salesman for Jumbo Corp. Last year his sales target was $3 million. This year in an aggressive bid for growth, the company increased the sales quota for all its sales people to $5 million, although the market for Jumbo Corps products had slowed. In order to meet his target, Bryan bribed an official of a potential customer. The roots of his unethical behavior can be traced to ___ cultural differences in countries. varying ethical standards in different nations. unrealistic performance goals. strong personal ethics among employees. Find the formula for the geometric sequence 1, 5, 25, 125, . Many individuals over the age of 40 develop intolerance for milk and milk-based products. A dairy has developed a line of lactose-free products that are more tolerable to such individuals. To assess the potential market for these products, the dairy commissioned a market research study of individuals over 40 years old in its sales area. A random sample of 250 individuals showed that 86 of them suffer from milk intolerance. Based on the sample results, calculate a 90% confidence interval for the population proportion that suffers milk intolerance. Interpret this confidence interval. a) First, show that it is okay to use the 1-proportion z-interval. b) Calculate by hand a 90% confidence interval. c) Provide an interpretation of your confidence interval. d) If the level of confidence was 95% instead of 90%, would the resulting interval be narrower or wider Ivan III was also called Ivan the Great because a. he helped to centralize the government by moving the capital to Moscow. c. he renewed trade with the west and opened Siberia for settlement. b. he built domed churches which inspired and awed the people. d. he was the first ruler of the independent state of Russia. Please help me fill in those couple blanks Im really struggling Suppose that a group of 10 people join a weight loss program for 3 months. Each person's weight is recorded at the beginning and at the end of the 3-month program. To test whether the weight loss program is effective, the data should be treated as: The Pueblo Revolt of 1680.A. Occurred because the Pueblos rejected everything Spanish,B. Resulted from the governors ignoring the Catholic priests' advice. C. Was a response to the Franciscan missionary demand that the Indians totally reject their traditional culture, D. Led to Spain's withdrawal from New Mexico for 100 years,